[personal profile] posic
Пусть А1 ← A2 ← A3 ← … -- проективная система абелевых групп. Первый (и единственный, кроме исходного функтора) производный функтор проективного предела limn1 An проективной системы (An) можно посчитать как коядро гомоморфизма

1−shift: ∏n=1 An → ∏n=1 An.

Ядром того же гомоморфизма является собственно проективный предел limn An.

Достаточным условием зануления производного функтора limn1 An является следующее условие Миттаг-Леффлера: если для любого m ≥ 1 убывающая последовательность подгрупп im pm,n: An → Am, n ≥ m, стабилизируется при n → ∞, то limn1 An = 0.

Является ли это условие необходимым для зануления группы limn1 An ? Нет, не является. Достаточно заметить, что производный функтор проективного предела, как и сам функтор проективного предела, коммутирует с операцией бесконечного произведения проективных систем абелевых групп.

Подобрав последовательность проективных систем абелевых групп, каждая из которых удовлетворяет условию Миттаг-Леффлера, но моменты стабилизации подгрупп-образов в каждой следующей проективной системе наступают позже, чем в предыдущей, и взяв прямое произведение такой бесконечной последовательности проективных систем, можно получить проективную систему абелевых групп, в которой стабилизация подгрупп-образов отображений проекции не наступает никогда, а производный функтор проективного предела этой проективной системы равен нулю, тем не менее.

Можно ли превратить условие Миттаг-Леффлера в необходимое и достаточное? Можно.

Теорема. Пусть A1 ← A2 ← A3 ← … -- проективная система абелевых групп. Рассмотрим бесконечную прямую сумму счетного числа копий проективной системы (An) и обозначим эту проективную систему через (Bn) = ⊕ω (An). Тогда следующие три условия эквивалентны:

- проективная система (An) удовлетворяет условию Миттаг-Леффлера;
- проективная система (Bn) удовлетворяет условию Миттаг-Леффлера;
- limn1 Bn = 0.

Кто-нибудь из читающих эти строки встречал такой результат где-нибудь?

P.S. Спросил на MathOverflow: https://mathoverflow.net/questions/270762/the-mittag-leffler-condition-as-necessary-and-sufficient
(will be screened)
(will be screened if not on Access List)
(will be screened if not on Access List)
If you don't have an account you can create one now.
HTML doesn't work in the subject.
More info about formatting

If you are unable to use this captcha for any reason, please contact us by email at support@dreamwidth.org

June 2025

S M T W T F S
1 2 3 4 56 7
8 9 10 1112 13 14
15161718192021
22232425262728
2930     

Most Popular Tags

Style Credit

Expand Cut Tags

No cut tags
Page generated Jun. 14th, 2025 09:21 pm
Powered by Dreamwidth Studios